Christian's maximum and minimum

Algebra Level 5

Let x x and y y be real numbers satisfying 4 x 2 + 5 y 2 = 1 4x^2 + 5y^2 = 1 . Over all such pairs, let the maximum and minimum values of 2 x 2 + 3 x y + 2 y 2 2x^2 + 3xy + 2y^2 be M M and N N respectively. If M + N + M N = a b M+N + MN = \frac{a}{b} , where a a and b b are coprime positive integers, what is the value of a + b a+b ?

This problem is posed by Christian L .


The answer is 159.

This section requires Javascript.
You are seeing this because something didn't load right. We suggest you, (a) try refreshing the page, (b) enabling javascript if it is disabled on your browser and, finally, (c) loading the non-javascript version of this page . We're sorry about the hassle.

18 solutions

C Lim
Jul 14, 2013

Substitute x = 1 2 cos t x = \frac 1 2 \cos t and y = 1 5 sin t y = \frac 1 {\sqrt 5} \sin t . The expression 2 x 2 + 3 x y + 2 y 2 2x^2 + 3xy + 2y^2 then becomes:

1 2 cos 2 t + 2 5 sin 2 t + 3 2 5 sin t cos t \frac 1 2 \cos^2 t + \frac 2 5 \sin^2 t + \frac 3 {2\sqrt 5} \sin t \cos t .

Using the double angle formulae cos 2 t = cos ( 2 t ) + 1 2 \cos^2 t = \frac{\cos(2t) + 1} 2 and sin t cos t = 1 2 sin 2 t \sin t\cos t = \frac 1 2 \sin 2t , the above expression becomes:

9 20 + cos 2 t 20 + 3 4 5 sin 2 t \frac 9 {20} + \frac{\cos 2t} {20} + \frac 3 {4\sqrt 5} \sin 2t

Now we apply the following trick.

Claim : The maximum and minimum values of A cos θ + B sin θ A\cos\theta + B\sin\theta are + A 2 + B 2 +\sqrt{A^2+B^2} and A 2 + B 2 -\sqrt{A^2+B^2} .

Proof :

By considering the point (A, B) on the circle with radius R = A 2 + B 2 R = \sqrt{A^2 + B^2} , we can write A = R cos α A = R\cos\alpha and B = R sin α B = R\sin\alpha . Then:

A cos θ + B sin θ = R cos α cos θ + R sin α sin θ = R cos ( θ α ) . A \cos\theta + B\sin\theta = R\cos\alpha \cos\theta + R\sin\alpha\sin\theta = R\cos(\theta-\alpha).

The maximum and minimum values of this expression are clearly +R and -R respectively. (QED)

Conclusion

The maximum and minimum values are M = 9 20 + 23 200 M=\frac 9{20} + \sqrt{\frac {23}{200}} and N = 9 20 23 200 N=\frac 9{20} - \sqrt{\frac{23}{200}} . From this, M+N+MN = 79/80.

Very elegant. I brute forced this via Lagrange multipliers, which took three pages of scratch work. Knew there must be a better way.

Andrew Edwards - 7 years, 11 months ago

One thing I would suggest: when you apply the half-angle formula, you don't note the manipulation applied to sin 2 t \sin^2 t , perhaps that would be worth a mention.

Andrew Edwards - 7 years, 11 months ago

Great solution, now it seems obvious why you did it because of a sum of squares. Very nice!

faraz masroor - 7 years, 11 months ago

This is so well explained! Nice work!

Akshat Jain - 7 years, 11 months ago

Exactly the same method but did wrong calculation :((

Akshay Sharma - 5 years, 4 months ago

What was your inspiration to substitute 1/2cos(t) and not just say, cos(t)? How did you think about substituting a trigonometric function?

Aradhya Kasera - 7 years, 10 months ago

Log in to reply

He set them up that way to satisfy 4x^2+5y^2=1.

Gino Pagano - 7 years, 10 months ago
Anqi Li
Dec 9, 2013

I see no obvious inequality method to solve this problem, so we can opt for Lagrange Multipliers . In my honest opinion, this is further motivated by the fact that we want both the extremum . This is what the Lagrange Multipliers say:

tagtext tagtext

Denote g ( x , y ) = 4 x 2 + 5 y 2 1 g(x,y) = 4x^2 + 5y^2 - 1 and f ( x , y ) = 2 x 2 + 3 x y + 2 y 2 f(x,y) = 2x^2 + 3xy + 2y^2 . So the candidates for the extremal values are the points for which g ( x , y ) = 0 \nabla g(x,y) = 0 and the solutions to the system:

  • f x = λ g x f_x = \lambda g_x

  • f y = λ g y f_y = \lambda g_y

  • g ( x , y ) = 0 g(x,y) = 0

Now firstly, notice that g = 8 x , 10 y \nabla g = \langle 8x,10y \rangle and f = 4 x + 3 y , 3 x + 4 y \nabla f = \langle 4x+3y, 3x+4y \rangle , we see that the only solution to g ( x , y ) = 0 \nabla g(x,y) = 0 is ( x , y ) = ( 0 , 0 ) (x,y) = (0,0) which does not satisfy the g ( x , y ) = 0 g(x,y) = 0 condition. Whence, we can safely conclude that the critical points are solutions of the system:

  • 4 x + 3 y = 8 λ x 4x+3y = 8 \lambda x

  • 3 x + 4 y = 10 λ y 3x + 4y = 10 \lambda y

  • 4 x 2 + 5 y 2 = 1 4x^2 + 5y^2 = 1

We isolate variables in the first 2 2 equations to get:

y = ( 8 λ 3 4 3 ) x y = ( \frac{8 \lambda}{3}-\frac{4}{3}) x

x = ( 10 λ 3 4 3 ) y x = ( \frac{10 \lambda}{3} - \frac{4}{3}) y

Substitute the first equation into the second and we have, x = ( 10 λ 3 4 3 ) ( 8 λ 3 4 3 ) x x = ( \frac{10 \lambda}{3} - \frac{4}{3}) ( \frac{8 \lambda}{3}-\frac{4}{3}) x so either x = 0 x = 0 or we have to solve the quadratic ( 10 λ 3 4 3 ) ( 8 λ 3 4 3 ) = 1 ( \frac{10 \lambda}{3} - \frac{4}{3}) ( \frac{8 \lambda}{3}-\frac{4}{3}) = 1 . The former is rejected because of inconsistency within the system.

As for the latter, we solve to get 2 2 answers, corresponding to the minimum and maximum respectively: λ = { 1 20 ( 9 + 46 ) , 1 20 ( 9 46 ) } \lambda = \{ \frac{1}{20} (9+ \sqrt{46}), \frac{1}{20} (9 - \sqrt{46}) \} .


For this final step, we have to notice the important factorisation that M + M N + N = ( M + 1 ) ( N + 1 ) 1 M + MN + N = (M+1)(N+1) - 1 . Joking, actually, we don't. The more important observation is that we have conjugates in our value of M , N M ,N , so directly adding and multiplying is actually a better idea. So, we have:

M + M N + N = 1 20 ( 9 + 46 ) + 1 20 ( 9 + 46 ) 1 20 ( 9 46 ) + 1 20 ( 9 46 ) = 18 20 + 81 46 400 = 79 80 M + MN + N = \frac{1}{20} (9+ \sqrt{46}) + \frac{1}{20} (9+ \sqrt{46})\frac{1}{20} (9 - \sqrt{46}) + \frac{1}{20} (9 - \sqrt{46}) = \frac{18}{20} + \frac{81 - 46}{400} = \frac{79}{80} .

In conclusion, the answer that we want is: a + b = 79 + 80 = 159 a+b = 79+80 = \fbox{159} .

You didn't have to appeal to Lagrange Multipliers. Since ( 2 x ) 2 + ( 5 y ) 2 = 1 (2x)^2+(\sqrt{5}y)^2= 1 , there must exist a real angle θ \theta such that cos θ = 2 x \cos \theta= 2x , sin θ = 5 y \sin \theta= \sqrt{5}y . The expression in question equals: 2 x 2 + 3 x y + 2 y 2 = cos 2 θ 2 + 3 2 5 sin θ cos θ + 2 sin 2 θ 5 = 9 20 + cos 2 θ 20 + 3 4 5 sin 2 θ = 9 20 + 46 20 ( 2 46 cos 2 θ + 3 × 20 4 5 × 46 sin 2 θ ) = 9 20 + 46 20 ( sin α cos 2 θ + cos α sin 2 θ ) = 9 20 + 46 20 sin ( α + 2 θ ) \begin{array}{lcl} 2x^2 +3xy + 2y^2&=&\frac{\cos^2 \theta}{2} + \frac{3}{2\sqrt{5}}\sin \theta \cos \theta + \frac{2\sin^2 \theta}{5} \\ & = & \frac{9}{20} + \frac{\cos 2\theta}{20} + \frac{3}{4\sqrt{5}}\sin 2\theta & \\ & = & \frac{9}{20} + \frac{\sqrt{46}}{20} \left ( \frac{2}{\sqrt{46}} \cos 2\theta + \frac{3 \times 20}{4\sqrt{5} \times \sqrt{46}} \sin2\theta \right ) \\ & = & \frac{9}{20} + \frac{\sqrt{46}}{20} \left (\sin \alpha \cos 2\theta + \cos \alpha \sin 2\theta \right ) \\ & = & \frac{9}{20} + \frac{\sqrt{46}}{20} \sin( \alpha + 2\theta ) \\ \end{array} where α = sin 1 ( 2 46 ) \alpha= \sin^{-1} \left ( \dfrac{2}{\sqrt{46}} \right ) . Optimizing the function shouldn't be hard now.

Sreejato Bhattacharya - 7 years, 6 months ago

Log in to reply

Yes, the trigonometric parametrization is probably the best approach to figure out exactly what is happening here.

Alexander Borisov - 7 years, 6 months ago

I am aware that the image is quite small so here's the link for those who want to view it: click me .

And there's a typo in the wording, I meant:

The former is rejected because of inconsistency with the system .

Anqi Li - 7 years, 6 months ago

While Lagrange Multiplies almost always work (but be sure that you cover all possibilities), they don't provide much insight into the problem. You could use this method to find the maximum and minimum points, and then justify it.

Actually, your factorization joke touches on a possible approach. The hint is to show that the set of possible values satisfy a certain given quadratic inequality, say g ( u ) 0 g(u) \geq 0 , then M + N + M N = ( M + 1 ) ( N + 1 ) = g ( 1 ) 1 M + N + MN = (M+1)(N+1) = g(-1) -1 .

Calvin Lin Staff - 7 years, 6 months ago

Its just an eqn of an ellipse and pair of st lines dont complicate , but anyways gr8 solution dude

Sai Arvind - 7 years, 6 months ago
Vô Tình
Jul 14, 2013

Let P = 2 x 2 + 3 x y + 2 y 2 P=2x^2+3xy+2y^2 .

If y = 0 = > x 2 = 1 4 , P = 1 2 y=0 => x^2=\frac{1}{4},P=\frac{1}{2}

If y 0 y \neq 0 Let t = x y t=\frac{x}{y} and P = 2 x 2 + 3 x y + 2 y 2 = 2 x 2 + 3 x y + 2 y 2 4 x 2 + 5 y 2 = 2 t 2 + 3 t + 2 4 t 2 + 5 P=2x^2+3xy+2y^2=\frac{2x^2+3xy+2y^2} {4x^2+5y^2}=\frac{2t^2+3t+2}{4t^2+5}

Note that 4 t 2 + 5 = 1 y 2 4t^2+5=\frac{1}{y^2} So t R t \in \mathbb{R}

Consider the function f ( t ) = 2 t 2 + 3 t + 2 4 t 2 + 5 ; t R f(t)=\frac{2t^2+3t+2}{4t^2+5}; t\in \mathbb{R}

f ( x ) = 12 t 2 + 4 t + 15 ( 4 t 2 + 5 ) 2 f'(x)=\frac{-12t^2+4t+15}{(4t^2+5)^2} ; f ( x ) = 0 x = 1 + 46 6 = a x = 1 46 6 = b f'(x)=0 \Leftrightarrow x=\frac{1+\sqrt{46}}{6}=a \vee x=\frac{1-\sqrt{46}}{6}=b

And so M = max P = max f ( x ) = f ( a ) = 9 + 46 20 ; N = min P = min f ( x ) = f ( b ) = 9 46 20 M=\max P=\max f(x)=f(a)=\frac{9+\sqrt{46}}{20}; N=\min P=\min f(x)= f(b)=\frac{9-\sqrt{46}}{20}

Then it's easy to calculate the sum M + N + M N = 79 80 M+N+MN=\frac{79}{80}

How do you justify that M=maxP ?

Gabriel Romon - 7 years, 11 months ago

Log in to reply

The limit of f ( x ) f(x) as x x goes to both infinities is 2 4 = 1 2 \frac{2}{4}=\frac{1}{2} , and those two values for x x ( a and b ) \left( a \text{ and }b\right) are the only extrema of f ( x ) f(x) , so those must be the maximum and the minimum.

Tim Vermeulen - 7 years, 10 months ago
Anubhav Singh
Jul 18, 2013

Let Q=2x^2+3xy+2y^2. If y=0=>x2=14,Q=12 If y≠0 Let t=xy and Q=2x^2+3xy+2y^2=(2x^2+3xy+2y^2)/(4x^2+5y^2)=(2t^2+3t+2)/(4t^2+5)

now 4t^2+5=1y^2 So t∈R

Consider the function f(t)=(2t^2+3t+2)/(4t^2+5);

Note that 4t^2+5=1/y^2 t∈R

f′(x)=(−12t^2+4t+15)/(4t^2+5)2;

f′(x)=0⇔x=(1+46)/√6=a∨x=(1−46)/√6=b

And so M=maxQ=maxf(x)=f(a)=(9+46)/√20;N=minQ=minf(x)=f(b)=(9−46)/√20

Then it's easy to calculate the sum M+N+MN=79/80

Let $y=kx$ then $x^2(4+5k^2)=1$ and $A=x^2(2+3k+2k^2)$ Denote $u=\frac{9+sqrt46}{20}$ and $v=\frac{9-sqrt46}{20}$ ( this number comes from solving later equation) Then it is easy to check that $v(4+5k^2)<=2+3k+2k^2<=u(4+5k^2)$ Then the maximum and minimum value of A is u and v. Thus u+v+uv=79/80. the answer is 159.

Let y = k x y=kx then x 2 ( 4 + 5 k 2 ) = 1 x^2(4+5k^2)=1 and A = x 2 ( 2 + 3 k + 2 k 2 ) A=x^2(2+3k+2k^2) Denote u = 9 + 46 20 u=\frac{9+\sqrt{46}}{20} and v = 9 46 20 v=\frac{9-\sqrt{46}}{20} ( this number comes from solving later equation) Then it is easy to check that v ( 4 + 5 k 2 ) < = 2 + 3 k + 2 k 2 < = u ( 4 + 5 k 2 ) v(4+5k^2)<=2+3k+2k^2<=u(4+5k^2) Then the maximum and minimum value of A is u and v. Thus u + v + u v = 79 / 80 u+v+uv=79/80 . the answer is 159.

Oscar Harmon - 7 years, 11 months ago

Log in to reply

thank you for rèformatting my solution!

Bờ La Bốc Khói - 7 years, 11 months ago
Nishant Sharma
Jan 3, 2014

We will attempt to convert the problem from two variables into one parameter. We can do so using the transformation ( x , y ) = ( cos θ 2 , sin θ 5 ) (x,y)=(\displaystyle\frac{\cos\theta}{2},\displaystyle\frac{\sin\theta}{\sqrt{5}}) .

Let us denote the given expression by P P .

Using the transformation,

P = cos 2 θ 2 + 3 sin θ cos θ 2 5 + 2 sin 2 θ 5 P=\displaystyle\frac{\cos^2\theta}{2}+\displaystyle\frac{3\sin\theta\cos\theta}{2\sqrt{5}}+\displaystyle\frac{2\sin^2\theta}{5}

or P = 3 sin 2 θ 4 5 + cos 2 θ 20 + 9 20 P=\displaystyle\frac{3\sin2\theta}{4\sqrt5}+\displaystyle\frac{\cos2\theta}{20}+\displaystyle\frac{9}{20} ( ( Using double-angle sin \sin and cos \cos formulae ) )

Let A = ( 3 4 5 ) 2 + ( 1 20 ) 2 A=\sqrt{\big(\displaystyle\frac{3}{4\sqrt5}\big)^2+\big(\displaystyle\frac{1}{20}\big)^2} .

Then M = A + 9 20 , N = A + 9 20 M=A+\displaystyle\frac{9}{20},\,N=\,-A+\displaystyle\frac{9}{20} .

So M + N + M N = 2 × 9 20 + ( 9 20 ) 2 A 2 = 79 80 M+N+MN=2\times\displaystyle\frac{9}{20}+(\displaystyle\frac{9}{20})^2-A^2=\displaystyle\frac{79}{80} .

Then a + b = 159 a+b=\boxed{159} .

sahiii be !! good

Sajan Kapil - 7 years, 4 months ago

Given that 4 x 2 + 5 y 2 = 1 4x^2 + 5y^2 = 1 ( 2 x ) 2 + ( 5 y ) 2 = 1 \implies (2x)^2 + (\sqrt 5 y)^2 = 1 , which is an equation for ellipse. We can substitute 2 x = cos θ 2x = \cos \theta and 5 y = sin θ \sqrt 5 y = \sin \theta and then:

2 x 2 + 3 x y + 2 y 2 = 1 2 cos 2 θ + 3 2 5 sin θ cos θ + 2 5 sin 2 θ = 1 4 ( 1 + cos 2 θ ) + 3 4 5 sin 2 θ + 1 5 ( 1 cos 2 θ ) = 9 20 + 1 20 cos 2 θ + 3 4 5 sin 2 θ = 9 20 + 46 20 sin ( 2 θ + tan 1 5 15 ) \begin{aligned} 2x^2 + 3xy + 2y^2 & = \frac 12 \cos^2 \theta + \frac 3{2\sqrt 5}\sin \theta \cos \theta + \frac 25 \sin^2 \theta \\ & = \frac 14 (1 + \cos 2 \theta) + \frac 3{4\sqrt 5}\sin 2 \theta + \frac 15 (1-\cos 2 \theta) \\ & = \frac 9{20} + \frac 1{20} \cos 2 \theta + \frac 3{4\sqrt 5}\sin 2 \theta \\ & = \frac 9{20} + \frac {\sqrt{46}}{20} \sin \left(2 \theta + \tan^{-1} \frac {\sqrt 5}{15} \right) \end{aligned}

Then we have:

{ M = max ( 9 20 + 46 20 sin ( 2 θ + tan 1 5 15 ) ) = 9 + 46 20 when sin ( 2 θ + tan 1 5 15 ) = 1 N = min ( 9 20 + 46 20 sin ( 2 θ + tan 1 5 15 ) ) = 9 46 20 when sin ( 2 θ + tan 1 5 15 ) = 1 \begin{cases} M = \max \left(\dfrac 9{20} + \dfrac {\sqrt{46}}{20} \sin \left(2 \theta + \tan^{-1} \dfrac {\sqrt 5}{15} \right) \right) = \dfrac {9+\sqrt{46}}{20} & \text{when } \sin \left(2 \theta + \tan^{-1} \dfrac {\sqrt 5}{15} \right) = 1 \\ N = \min \left(\dfrac 9{20} + \dfrac {\sqrt{46}}{20} \sin \left(2 \theta + \tan^{-1} \dfrac {\sqrt 5}{15} \right) \right) = \dfrac {9-\sqrt{46}}{20} & \text{when } \sin \left(2 \theta + \tan^{-1} \dfrac {\sqrt 5}{15} \right) = -1 \end{cases}

Therefore, M + N + M N = 9 + 46 20 + 9 46 20 + 9 + 46 20 × 9 46 20 = 9 10 + 7 80 = 79 80 M+N+MN = \dfrac {9+\sqrt{46}}{20} + \dfrac {9-\sqrt{46}}{20} + \dfrac {9+\sqrt{46}}{20} \times \dfrac {9-\sqrt{46}}{20} = \dfrac 9{10} + \dfrac 7{80} = \dfrac {79}{80} a + b = 79 + 80 = 159 \implies a+b = 79+80 = \boxed{159} .

In the last and second last lines, shouldn't you replace all the 49 \sqrt {49} s with 46 \sqrt {46} s? Excellent, short solution.

Harsh Poonia - 2 years, 3 months ago

Log in to reply

Thanks. I have changed them.

Chew-Seong Cheong - 2 years, 3 months ago
Ricardo Alencar
Jul 19, 2013

Knowing that 4 x 2 + 5 y 2 = 1 4x^2 + 5y^2 = 1 representes the equation of a ellipsis, is convenient to make the following trigonometric transformations: $$ x = \frac{\cos\theta}{2}, y = \frac{\sin\theta}{\sqrt{5}} $$ Onto the second equation: $$ f(\theta) = 2\cdot\frac{\cos^2\theta}{4} + 3\cdot\frac{\cos\theta}{2}\cdot\frac{\sin\theta}{\sqrt{5}} + 2\cdot\frac{\sin^2\theta}{5} $$ Using double arc transformation and some algebraic manipulation we'll get to: $$ f(\theta) = \frac{1}{20}\cdot\cos(2\theta) + \frac{3}{4\sqrt{5}}\cdot\sin(2\theta) + \frac{9}{20} $$ As we can see, it's always possible to do the following transformation: $$ A\cos x + B\sin x = \sqrt{A^2 + B^2}\left(\frac{A}{\sqrt{A^2 + B^2}}\cos x + \frac{B}{\sqrt{A^2 + B^2}}\sin x\right) $$ It's even possible to see that: $$ A\cos x + B\sin x = \sqrt{A^2 + B^2}\sin(x + \alpha) $$ Where α \alpha is obtained contructing a right triangle of sides A A and B B . Finally, we observe that the maximum and minimum values of f ( θ ) f(\theta) occur when sin ( α + x ) = ± 1 \sin(\alpha + x) = \pm 1 . Hence: $$ M =\frac{9}{20} + \sqrt{\frac{1}{400} + \frac{9}{80}}, N =\frac{9}{20} - \sqrt{\frac{1}{400} + \frac{9}{80}} $$ Calculating M + N + M N M + N + MN we come to the answer: $$ M + N + MN = \frac{79}{80} \Rightarrow a + b = 159 $$

Arnav Shringi
Jul 16, 2013

M = .789116 N = .110884 MN = .087500

M+N+MN = .987500 = 79/80 a+b=79+80 =159

It is unclear to me how a decimal approximation can be used to obtain an exact rational value.

Calvin Lin Staff - 7 years, 11 months ago
Sai Arvind
Dec 11, 2013

This is very simple ,start off by noticing that 4x^2+5y^2=1 is an equation of an ellipse and the reqn eqn is the equation of a pair of straight line. Now you can use the above to simply solve the prob or u can also use lagrange's theorem . I think ull get M=1/20(9+4root 6 ) and N to be its conjugate

small error sry its root of (46) not 4 root (6 )

Sai Arvind - 7 years, 6 months ago
Ang Yan Sheng
Jul 20, 2013

Let F ( x , y ) = 2 x 2 + 3 x y + 2 y 2 F(x,y)=2x^2+3xy+2y^2 . We shall consider the following quadratic, where λ \lambda is a real number: F ( x , y ) λ = ( 2 4 λ ) x 2 + 3 x y + ( 2 5 λ ) y 2 F(x,y)-\lambda=(2-4\lambda)x^2+3xy+(2-5\lambda)y^2 We want to find out when we can write this polynomial as ± ( a x + b y ) 2 \pm(ax+by)^2 ; this happens when the discriminant of the quadratic is 0. In other words, 3 2 4 ( 2 4 λ ) ( 2 5 λ ) = 0 80 λ 2 72 λ + 7 = 0 λ = 9 ± 46 20 \begin{aligned} 3^2-4(2-4\lambda)(2-5\lambda)&=0\\ 80\lambda^2-72\lambda+7&=0\\ \lambda&=\frac{9\pm\sqrt{46}}{20} \end{aligned} When λ = 9 + 46 20 \lambda=\frac{9+\sqrt{46}}{20} , we have F ( x , y ) λ = 1 46 5 x 2 + 3 x y + 1 46 4 = ( 46 1 5 x + 46 + 1 4 y ) 2 0 \begin{aligned} F(x,y)-\lambda&=\frac{1-\sqrt{46}}5x^2+3xy+\frac{-1-\sqrt{46}}4\\ &=-\left(\sqrt{\frac{\sqrt{46}-1}5}x+\sqrt{\frac{\sqrt{46}+1}4}y\right)^2\\ &\leq0\end{aligned} Hence F ( x , y ) 9 + 46 20 F(x,y)\leq\frac{9+\sqrt{46}}{20} , and equality holds if and only if 46 1 5 x + 46 + 1 4 y = 0 \sqrt{\frac{\sqrt{46}-1}5}x+\sqrt{\frac{\sqrt{46}+1}4}y=0 and 4 x 2 + 5 y 2 = 1 4x^2+5y^2=1 . Notice that the first equation describes a line passing through the origin, and the second describes an ellipse centred at the origin. Hence these two simultaneous equations indeed has a solution, and thus there exists x , y x,y such that F ( x , y ) = 9 + 46 20 F(x,y)=\frac{9+\sqrt{46}}{20} . Therefore M = 9 + 46 20 \boxed{M=\frac{9+\sqrt{46}}{20}} .

Similarly, when λ = 9 46 20 \lambda=\frac{9-\sqrt{46}}{20} , we have F ( x , y ) λ = 1 + 46 5 x 2 + 3 x y + 1 + 46 4 = ( 1 + 46 5 x + 1 + 46 4 y ) 2 0 \begin{aligned} F(x,y)-\lambda&=\frac{1+\sqrt{46}}5x^2+3xy+\frac{-1+\sqrt{46}}4\\ &=\left(\sqrt{\frac{1+\sqrt{46}}5}x+\sqrt{\frac{-1+\sqrt{46}}4}y\right)^2\\ &\geq0\end{aligned} Hence F ( x , y ) 9 46 20 F(x,y)\geq\frac{9-\sqrt{46}}{20} . By a similar argument to the above, equality is possible. Hence N = 9 46 20 \boxed{N=\frac{9-\sqrt{46}}{20}} .

Now M M and N N are the roots of the quadratic equation 80 λ 2 72 λ + 7 = 0 80\lambda^2-72\lambda+7=0 , so by Vieta's formulas we get M + N = 72 80 M N = 7 80 M + N + M N = 79 80 , \begin{aligned}M+N&=\frac{72}{80}\\MN&=\frac{7}{80}\\\therefore M+N+MN&=\frac{79}{80},\end{aligned} and thus the answer is 79 + 80 = 159 79+80=\boxed{159} .

Kamran Azmat
Jul 16, 2013

M = .789116 N = .110884 MN = .087500

M+N+MN = .987500 = 79/80

Finn Hulse
Apr 24, 2014

Nice...

Tửng Khùng
Jul 20, 2013

4x^2+5y^2=1 (1)

A=2x^2 + 3xy + 2y^2 (2)

if y = 0 then x^2= 1/4 => A=1/2

y not equals to 0

(1) <=> 1/y^2= 4 x^2/y^2 + 5

(2) <=> A/y^2= 2 x^2/y^2 + 3x/y +2

<=> A*(4 x^2/y^2 +5)=2 x^2/y^2 + 3x/y +2

<=> (4A-2) x^2/y^2 - 3 x/y +5A -2 =0

9 - 4 (4A-2) (5A-2) >=0

<=> 80A^2 - 72A + 7 <=0 <=> N=.....<= A <= M=....

M+N+MN= 72/80 + 7/80 = 79/80 => a+b = 159 ( N=< A=1/2<=M )

Benson Li
Jul 19, 2013

let x and y be described by the parametrization of x=.5cos(t), y=(sin (t))/(sqrt5) This will satisfy the requirement of 4x^{2}+5y^{2}=1. Plugging those into our equation, it now becomes .5(cos(t))^2 + .75/(sqrt5) (sin(2t))+.4(sin(t))^2. Further simplification yields .1(cos (t))^2 + .75/(sqrt5) (sin(2t))+.4. Now, taking the derivative and setting it to zero, we see than tan(2t)=3 sqrt5. That means the maximum and minimum occur when t is in quadrant 1 and quadrant three. In the parametrized equation, the only term that depends on this change is .75/(sqrt5) (sin(2t)). From tan(2t), we can get sin(2t) and cos(2t) from drawing up a triangle, and from there we can get (cos(t))^2 by using the half angle formula. Also, we can rewrite M+N+MN as (M+1)(N+1)-1. Plugging in the values for sin(2t) and (cos(t))^2 will yield the maximum and plugging in -sin(2t) and (cos(t))^2 will yield the minimum. Finally, plugging these values into (M+1)(N+1)-1, we get 79/80. So the answer is 159.

Let Q=2x^2+3xy+2y^2. If y=0=>x2=14,Q=12 If y≠0 Let t=xy and Q=2x^2+3xy+2y^2=(2x^2+3xy+2y^2)/(4x^2+5y^2)=(2t^2+3t+2)/(4t^2+5)

now 4t^2+5=1y^2 So t∈R

Consider the function f(t)=(2t^2+3t+2)/(4t^2+5);

Note that 4t^2+5=1/y^2 t∈R

f′(x)=(−12t^2+4t+15)/(4t^2+5)2;

f′(x)=0⇔x=(1+46)/√6=a∨x=(1−46)/√6=b

And so M=maxQ=maxf(x)=f(a)=(9+46)/√20;N=minQ=minf(x)=f(b)=(9−46)/√20

Then it's easy to calculate the sum M+N+MN=79/80

Abhishek Ranjan
Jul 17, 2013
Harsa Mitra
Jul 16, 2013

Let Q=2x^2+3xy+2y^2. If y=0=>x2=14,Q=12 If y≠0 Let t=xy and Q=2x^2+3xy+2y^2=(2x^2+3xy+2y^2)/(4x^2+5y^2)=(2t^2+3t+2)/(4t^2+5)

now 4t^2+5=1y^2 So t∈R

Consider the function f(t)=(2t^2+3t+2)/(4t^2+5);

Note that 4t^2+5=1/y^2 t∈R

f′(x)=(−12t^2+4t+15)/(4t^2+5)2;

f′(x)=0⇔x=(1+46)/√6=a∨x=(1−46)/√6=b

And so M=maxQ=maxf(x)=f(a)=(9+46)/√20;N=minQ=minf(x)=f(b)=(9−46)/√20

Then it's easy to calculate the sum M+N+MN=7980

Don't copy solutions that have already been submitted.

Tim Vermeulen - 7 years, 11 months ago

Log in to reply

Hm, actually people are copying Harsa's solutions, not the other way around. Look at the post timings.

Ahaan Rungta - 7 years, 10 months ago

Log in to reply

I wasn't talking about the solutions of Abhishek and Anubhav, in fact, you should look at the timestamps, as their solutions are more recent than my comment. ;-) I was talking about the solution by Vô, which uses the exact same wordings and was posted before this solution.

Tim Vermeulen - 7 years, 10 months ago

0 pending reports

×

Problem Loading...

Note Loading...

Set Loading...